You are on page 1of 22

Test Information

Test Name JEE(Main + Advanced) Enthusiast Score-I (J1-1604) Paper-1 Total Questions 54

JEE
Test Stream JEE MAIN + ADVANCED Test Pattern
(Advanced)

Total Marks 198 Duration 180 minutes

Paper Code 1001CJA102120132

Test Question Language:- English Test Instruction

Section: PHYSICS SEC-1 PHYSICS SEC-1 Instruction

1. Atoms of a hydrogen like gas are in a particular excited energy level. When these atoms de-excite, they emit photons of different energies.
Maximum and minimum energies of emitted photons are Emax = 52.224 eV and Emin = 1.224 eV respectively. Calculate principal quantum number
of initially excited energy level. (Ionisation energy of hydrogen atom is 13.6 eV)

a. 6

b. 4

c. 5

d. 3

Solution. Let principal quantum number of initially excited energy level be n. Photon of maximum energy is emitted during transitions n → 1.
hence, Emax = (En – E1) …(1)
Since, difference between energy of two consecutive levels decreases when n increases, therefore, photon of minimum energy is emitted during
transition n → (n – 1)
hence, Emin = En – E(n – 1) …(2)
But energy of nth level is given by,

∴ from equation (1), …(3)

and from equation (3), …(4)

solving equation (3) and (4), and E1 = –54.4 eV and n = 5


Hence, the gas is singly ionized Helium.
∴ (C)

Answer. c

Right Marks: 3.00

Negative Marks: 1.00

2. An infinitely long metal cylinder of radius R and surface charge density σ is placed symmetrically with an imaginary surface of the shape of a
prism. The length of prism is L and its 3 sides are all equal to 3R. The flux through the prism is :

a.

b.

c.

d.

Solution. The area of cylinder inside prism is


hence charge inside is
Answer. c

Right Marks: 3.00

Negative Marks: 1.00

3. A rough ball is thrown such that it rotates clockwise and moves right in viscous air. The force due to pressure difference of air acts :

a. Upwards

b. Downwards

c. Nowhere

d. Can’t say

Solution. As air rotates with the ball and also air comes from front relative velocity is less above and more below hence pressure is high above and
low below causing the net force to be downwards.

Answer. b

Right Marks: 3.00

Negative Marks: 1.00

4. Mercury is filled in a glass beaker which is heated by a flame from below. The level of mercury with time

a. rises

b. falls

c. rises & then falls

d. falls & then rises

Solution. First glass receives heat thus it expands and then mercury expands more than glass.

Answer. d

Right Marks: 3.00

Negative Marks: 1.00

5. A bar magnet of mass m is given an initial velocity V0 towards the centre of a metallic coil of same mass initially at rest in gravity free space. The

total loss of energy is :

a.

b.

c.

d. cannot be determined

Solution. Energy loss =

Answer. d

Right Marks: 3.00


Negative Marks: 1.00

6. A string is wound on a spool which is kept on a rough horizontal floor having sufficient friction to prevent slipping. If string is pulled then

(1) String winds

(2) String unwinds


(3) Spool moves left
(4) Spool moves right
(5) Point P moves left
(6) point P moves right
Which of the above options are correct ?

a. 2,3,5

b. 1,4,6

c. 2, 4, 6

d. 2, 3, 4

Solution. Taking torque about pt of contact


⇒ torque on spool is clockwise, hence spool turns clockwise,
and goes right winding the string

Answer. b

Right Marks: 3.00

Negative Marks: 1.00

Section: PHYSICS SEC-2 PHYSICS SEC-2 Instruction

7. Two girls of equal mass m jump off from the border line of a stationary carriage of mass M with same horizontal velocity u relative to the
carriage in same direction. Neglect the effect of friction. Mark incorrect options.

a. They will impart greater velocity to the carriage by jumping off simultaneously

b. They will impart greater velocity to the carriage by jumping one after the other

c. They will impart same velocity to the carriage in whatever manner they jump off

d. Insufficient data to reply

Solution. When girls jump one by one total velocity of the system will be more. (because from conservation of momentum, velocity of car in this
case will be

instead of
when they jump simultaneously.
∴ (B, C, D)

Answer. a,c,d

Right Marks: 4.00

Negative Marks: 2.00

8. A body is suspended from a spring balance kept in a satellite. The reading of the balance is W1 when the satellite goes in an orbit of radius R and
is W2 when it goes in an orbit of radius 2R. Mark incorrect options.

a. W1 = W2

b. W1 < W2

c. W1 > W2

d. W1 ≠ W2

Answer. b,c,d

Right Marks: 4.00


Negative Marks: 2.00

9. Electric field intensity at a point (x, y) is given by where all parameters are in S.I. units. Then choose the correct
statement(s) out of the following :

a. It represents a conservative electric field.

b. It represents a non-conservative electric field.

c. Electric potential at point (1m, 1m) is –4V if electric potential at origin is zero.

d. Electric potential at point (1m, 1m) cannot be defined.

Solution.

It could be directly integrated without any reference to a path


⇒ field is conservative.
Integrating,
V = 0 at (0, 0) ⇒ C = 0
V = 2x3 – 6x2 y3 at (1m, 1m), V = –4V
∴ (a, c)

Answer. a,c

Right Marks: 4.00

Negative Marks: 2.00

10. For the circuit shown in figure, which of the following statements are correct ?

a. Time constant of the circuit is 0.5 sec.

b. In steady state current through battery is 1.0A.

c. In steady state current through inductor is 1.0A.

d. In steady state potential difference across 2Ω resistor is 4V.

Solution. Req. =

Time constant
in steady state, current through cell

voltage across 2Ω resistor = 2V


∴ (A, B, C)

Answer. a,b,c

Right Marks: 4.00

Negative Marks: 2.00

11. A solid cylinder of radius R is projected with velocity v0 on a rough horizontal surface of coefficient of friction ‘μ’ at t = 0 and at the same instant

it has an angular velocity of as shown in figure. Then


a. The cylinder stops at and then turns back.

b. At the cylinder starts pure rolling.

c. At angular velocity of the cylinder is .

d. At speed of the cylinder is v0.

Solution. The friction acts backwards initially as point of contact has velocity in forward direction and when the cylinder stops, the friction pulls it
back.
∴ (A, B, C)

Answer. a,b,c

Right Marks: 4.00

Negative Marks: 2.00

12. In a cylindrical region of radius R, there exists a time varying magnetic field B such that . A charged particle having charge q is
placed at the point P at a distance d (> R) from its centre O. Now, the particle is moved in the direction perpendicular to OP (see figure) by an

external agent upto infinity so that there is no gain in kinetic energy of the charged particle. Choose the correct statement/s.

a. Work done by external agent is if d = 2R

b. Work done by external agent is if d = 4R

c. Work done by external agent is if d = 4R

d. Work done by external agent is if d = 6R

Solution.

∴ (A), (C) and (D)

Answer. a,c,d

Right Marks: 4.00

Negative Marks: 2.00

Section: PHYSICS SEC-3 PHYSICS SEC-3 Instruction

13. A force , (where K is a +ve constant) acts on a particle moving in xy plane starting from origin, the particle is taken along the
positive x-axis to the point (a, 0) and then parallel to y axis to the point (a, a). If the total work done by force F on the particle is nka2, find n + 3.
Solution. Work done

then and

∴ W = W1 + W2 = – ka2
Hence n is –1, therefore n + 3 is 2.
Answer is 2.

Answer. 2.00

Right Marks: 4.00

Negative Marks: 0.00

14. A mercury thermometer reads 80ºC when the mercury is at 5.2cm mark and 60ºC when the mercury is at 3.9 cm mark. If the temperature when
the mercury level is at 2.6 cm mark is T (in degree Celsius), find T – 35

Solution.

or
1.3 (80 – T3) = 2.6 (20) or T3 = 40°C
Answer is 5

Answer. 5.00

Right Marks: 4.00

Negative Marks: 0.00

15. A standing wave with a frequency of 1000 hz in a column of mixture of unknown gases at 27°C produces node that are 0.1 m apart. Find the
value of r = for the mixture. Use R = 8.3 J/mol–K and molar mass of mixture 124.5 gm/mol. CP & CV are the molar heat capacities at constant
pressure and constant volume respectively.

Solution. T = (273 + 27) k = 300 k


ν = 1000 hz
λ = 0.2 m
M = 124.5 gm/mol
Wave velocity V = νλ = 200 m/s
γ=2

Answer. 2.00

Right Marks: 4.00

Negative Marks: 0.00

16. When a source of sound of frequency 600 Hz moves with velocity 30 m/s towards a stationary observer, the apparent wavelength is λ. If the
same source is kept stationary and the observer is moved with same velocity 30 m/s towards the source, then apparent wavelength is λ', then find
λ' – λ (in cm) for speed of sound in air to be 340 m/s.

Solution. In first case

In second case

So,
So, λ' – λ = 5 cm.

Answer. 5.00

Right Marks: 4.00

Negative Marks: 0.00

17. Two rigid rods of same material and of length L and are placed in equilibrium on a smooth horizontal plane at temperature T as shown in

figure. Springs are in their natural length. If the temperature is increased by ΔT amount energy stored in springs is , where k is the
spring constant of spring (A) and α is the coefficient of linear expansion of material. (Note: Neglect the change in spring constant due to increase in

temperature).

Solution. Total increase in length of rods

Let the compression in spring A is xA, B is xB and C is xC

and

Energy stored

⇒β=4

Answer. 4.00

Right Marks: 4.00

Negative Marks: 0.00

18. A particle is moving in a plane according to the law x = at2, y = bt3 where a = 2m/s2, b = 1 m/s3. Radius of curvature of particle at t = 1 s is found
to be . Find η.

Solution.

At t = 1 s

If radius of curvature is R then

v = 5 m/s

= 12 k

Answer. 5.00

Right Marks: 4.00

Negative Marks: 0.00

Section: CHEMISTRY SEC-1 CHEMISTRY SEC-1 Instruction

19. Which of the statement is incorrect?


a. Hyponitrous acid is a weak acid and it is isomeric with nitramide.

b. Sodium dihydrogen pyrophosphate does not exist.

c. [Co(en)2SO4]⊕ is a low spin optically active compound.

d. Acidic character (w.r.t. oxide acceptor) : XeF6 > XeO2F4 > XeO3

Answer. b

Right Marks: 3.00

Negative Marks: 1.00

20. Born Haber cycle helps to determine lattice enthalpy of ionic compound as well as electron gain enthalpy of different elements. Based on the

diagram drawn, answer the questions given below. What is the magnitude of lattice enthalpy of MgCl2 ?

a. 20kJ

b. 40kJ

c. 15kJ

d. 60kJ

Answer. a

Right Marks: 3.00

Negative Marks: 1.00

21. Which of the following statement(s) is(are) correct? (1) When a chalk stick is dipped in ink, both absorption and adsorption takes place and
surface of chalk retains the color of ink while the inside in white. (2) Water vapours are absorbed by silica gel
(3) Water vapours are adsorbed by anhydrous CaCl2
(4) The precipitate of Mg(OH)2 attains pink colour when precipitated in presence of magneson reagent

a. 4 only

b. 1 Only

c. 2, 3 only

d. 1, 2, 3 only

Solution. When a chalk is dipped in ink, the surface retains the colour of ink due to adsorption of coloured molecules while solvent of ink goes
deeper into stick due to absorption.
→ Precipitate of Mg(OH)2 attains blue colour when precipitated in the presence of magneson reagent.

Answer. b

Right Marks: 3.00

Negative Marks: 1.00

22. Density of a Vander Waal's gas at 500 K and 1 atm is found to be 0.8 kg/m3. It is also found that the gas diffuses times slower than pure O2
gas under identical conditions. The value of 'z' for the real gas is (R = 0.08 atm-ℓ/mol-K) :

a. 2/3

b. 3/2

c. 1.8
d. 0.8

Solution. d = 0.8 g/L


rgas =

or, = ; M = Mgas

or, ⇒ M = 48g

d= ⇒ Vm =

Z= =

Answer. b

Right Marks: 3.00

Negative Marks: 1.00

23. Which of the following oxyanion has 3 identical resonating structures?

a.

b.

c. ClO4–

d.

Answer. a

Right Marks: 3.00

Negative Marks: 1.00

24. (+)2–butanol has [θ]λ25 = 13.9°. A sample of 2-butanol containing both the enantiomers was found to have specific rotation value of –3.5° under
similar condition. The percentage of the (+) and (–) enantiomer present in the sample are, respectively :

a. 37.4% and 62.6%

b. 62.6% and 37.4%

c. 42.2% and 57.8%

d. 35.5% and 64.5%

Solution. O.P. ⇒ – × 100 ⇒ (–) 25.18%

Total d ⇒ 37.4
Total ⇒ 37.4 + 25.18 = 62.6

Answer. a

Right Marks: 3.00

Negative Marks: 1.00

Section: CHEMISTRY SEC-2 CHEMISTRY SEC-2 Instruction

25. Which of the following do NOT represent the correct sequence of indicated properties ?

a. [FeF6]3– > [NiF6]4– : Magnetic moment


b. [Fe(CN)6]3– < [Fe(NH3)6]3+ : stability constant

c. NiO > CaO > BeO : Basic character

d. F2 < Cl2 < Br2 < I2 : Colour intensity

Solution. (A) Fe3+ → 5 unpaired electron Ni2+ → 2 unpaired electron


(B) [Fe(CN)6]3– is more stable
(C) Across a period basic character ↓ and down the group, basic character ­.
(D) I2 is more polarisable.

Answer. b,c

Right Marks: 4.00

Negative Marks: 2.00

26. Which of the following is/are true statements?

a. In Puddling process wrought iron is produced.

b. In blast furnace phosphate and silicate are reduced into P and Si which pass into molten iron.

c. B, Ga, In, Ge and Si are refined by zone refining.

d. In Mond's process high spin complex is formed.

Answer. a,b,c

Right Marks: 4.00

Negative Marks: 2.00

27. If an optically active compound 'A' decompose through given parallel 1st order kinetics.

Initial mole of A is 2 ( k1 = 0.0693 sec–1 , k2 = 0.1386 sec–1). If only A, B & D are optically active compounds and their angle of rotation per mole are
60º, 30º, –90º respectively,
then which of the following is/are correct -

a. Angle of rotation after time 3.33 sec is 20º

b. Mixture is levorotatary after 75% decomposition of A

c. Overall t1/2 of A is 10/3 sec.

d. Percentage of C formed in the product is 11.11%

Answer. a,b,c,d

Right Marks: 4.00

Negative Marks: 2.00

28. 100 ml, 0.3 M solution of HCl is mixed with 50 of water to form solution (A). 50 ml of solution (A) is mixed with 50 ml, 0.05M NaOH to form
solution (B). Another 50 ml of solution (A) is mixed with 100 ml, 0.05M NH4OH to form solution (C). Last 50 ml of solution (A) is mixed with 100 ml,
0.1 M of AgNO3 to form solution (D),
[Given Ksp(AgCl) = 10–10 , Kb (NH4OH) = 2 × 10–5 , log 2 = 0.3, log 3 = 0.48)]

a. pH of solution (A) is 0.7

b. pH of solution (B) is 1.12

c. pH of solution (C) is 4.7

d. mole of Ag+ ion in solution D is 1.5 × 10–6 mole

Solution. A → 0.3 M, 100 ml HCl + 50 ml water


[H+] =
pH = 1 – log2 = 1 – 0.3 = 0.7
B → 50 ml, 0.2 M HCl + 50ml 0.05 M NaOH
HCl = 10 mmole, NaOH = 2.5 mmole
H+ =
pH = 2 – log 7.5 = 2 – 0.88 = 1.12
C → 50 ml, 0.2 M HCl + 100ml 0.05 M NH4OH
HCl + NH4OH → NH4Cl + H2O
10 mmole 5 mmole
5 mmole 0 5 mmole
[H+] =

pH =
= 1.48
D → 50 ml, 0.2 M HCl + 100ml 0.1 M AgNO3
HCl + AgNO3 → AgCl + HNO3
10 mmole 10 mmole
0 0 10 mmole 10 mmole

10–10 = [Ag+]2 [∵ [Ag+] = [Cl–] ]


or, [Ag+] = 10–5 M
∴ moles of Ag+ = 150 × 10–5 × 10–3 mole
= 1.5 × 10–6 mole

Answer. a,b,d

Right Marks: 4.00

Negative Marks: 2.00

29. Which of the following reactions give para bromophenol as a major product?

a.

b.

c.

d.

Answer. a,b,c

Right Marks: 4.00

Negative Marks: 2.00

30. In which of the following reaction(s) the product is/are correctly matched?

a. CH3–CH=O CH3–COOH

b.

c.
d.

Solution. (C)

(D)

Answer. a,b

Right Marks: 4.00

Negative Marks: 2.00

Section: CHEMISTRY SEC-3 CHEMISTRY SEC-3 Instruction

31. Among the following cations(s) :


Hg2+, Mg2+, Cu2+, Ag+, Pb2+
(i) Number of cation(s) gives(s) ppt with H2S / H+ is = (X)
(ii) Number of cation(s) gives(s) ppt with KI is = (Y)
(iii) Number of cation(s) gives(s) ppt with dil. HCl is = (Z)
Find the value of (X + Y + Z)

Answer. 10.00

Right Marks: 4.00

Negative Marks: 0.00

32. Consider the following reactions


(i) PbS + H2O2 →
(ii) Ag2O + H2O2 →
(iii) K4[Fe(CN)6] + H2O2 + H2SO4 →
(iv) K3[Fe(CN)6] + H2O2 + KOH →
(v) K2Cr2O7 + H2O2 + H2SO4

(vi) CrCl3 + H2O2 + KOH →


(vii) Cl2 + H2O2
(viii) KIO4 + H2O2
(ix) NaMnO4 + H2O2 + H2SO4 →
In how many reactions oxidising nature of H2O2 is observed.

Solution. 3

Answer. 3.00

Right Marks: 4.00

Negative Marks: 0.00

33. A definite amount of an ideal gas is taken from state-I to state-II isothermally at 300K. If the process is carried out reversibly then work done is –
1750 kJ and if process is carried out irreversibly then work done is –1000 kJ. If ΔStotal is x kJ/K for irreversible isothermal process then give 'x' .

Solution. W = –1750 kJ
Wirr = –1000 kJ
ΔStotal =

= = =

= 2.5 kJK–1
Answer. 2.50

Right Marks: 4.00

Negative Marks: 0.00

34. Liquids A and B form ideal solution over the entire range of composition. At temperature T, equimolar binary solution of liquids A and B has
vapour pressure 45 Torr. At the same temperature, a new solution of A and B having mole fractions xA and xB respectively, has vapour pressure of
22.5 Torr. If the value of in the new solution is Y. Then the value ‘Y’ is :
(Given that the vapour pressure of pure liquid A is 20 Torr at temperature T)

Solution. Equimolar, PT = 45 torr


xA & xB ,
,

Initially , xA = xB = 0.5 ,
PT =
or, 45 =
or,
or, = 90 – 20 = 70 torr.

or, 22.5 = (20 –70) xA + 70


or, 22.5 = – 50 xA + 70
or, 50 xA = 47.5
or, xA =
∴ xB = 0.05
∴Y=

Answer. 19.00

Right Marks: 4.00

Negative Marks: 0.00

35. Consider the figure

Amino acids pKa values

α-COOH Side chain

Arginine 2.17 9.04 12.48


Alanine 2.34 9.69 ........
Aspartic acid 2.09 9.82 3.86
Lysine 2.18 8.95 10.79
Glutamic acid 2.19 9.67 4.25
Valine 2.32 9.62 ........

Calculate :
Number of amino acids move towards cathode = X
Number of non essential amino acids move towards anode = Y
Find =?

Solution. X = 4, Y = 2
X ⇒ Valine (PI = 5.97), Arginine (PI = 10.76), Lysine (PI = 9.87), Alanine (PI = 6.02)
Y ⇒ Glutamic acid (PI = 3.22), Aspartic acid (PI = 2.98)
PI =

Amino acid having PI > pH moves towards cathode


Amino acid having PI < pH moves towards anode
Answer. 2.00

Right Marks: 4.00

Negative Marks: 0.00

36. How many reagents can be used to distinguish these compounds :

(i) NaHCO3 (ii) Na metal (iii) Neutral FeCl3


(iv) AgNO3 + NH4OH (v) CS2, HgCl2 (vi) NaOI
(vii) 2,4–DNP

Solution.
Compound-I Compound-II

NaHCO3 ×

Na Metal

Neutral FeCl3 ×

AgNO3 + NH4OH × ×

CS2, HgCl2

NaOI × ×
2,4–DNP × ×

Answer. 2.00

Right Marks: 4.00

Negative Marks: 0.00

Section: MATHEMATICS SEC-1 MATHEMATICS SEC-1 Instruction

37. If where z > 0, then |(1 + z)f '(z) cot(f(z))| is equal to

a. 0

b. 1

c. 2

d. 3

Solution. differentiate both sides w.r.t. 'z'

sinf(z)(1 – z) = (2z + 1)sin(f(z)) + (z2 + z)cos(f(z))f '(z)


–3zsinf(z) = (z2 + z)cos(f(z))f '(z)
–3 = (z + 1)cot(f(z)).f '(z)

Answer. d

Right Marks: 3.00

Negative Marks: 1.00

38. Given vectors ; and . The volume of parallelopiped with three conterminous edges as

equals
a.

b.

c.

d.

Solution.

Answer. c

Right Marks: 3.00

Negative Marks: 1.00

39. A parabola whose focus S(3, 4) is touching the co-ordinate axes. The equation of circle whose diameter is the portion of tangent at vertex of the
parabola between the co-ordinate axes is

a. x2 + y2 – 6x – 8y = 0

b. x2 + y2 + 3x + 4y = 0

c. x2 + y2 + 6x + 8y = 0

d. x2 + y2 – 3x – 4y = 0

Solution. Required circle is the circle described on O(0,0) & S(3,4) as diameter
(x – 0)(x – 3) + (y – 0)(y – 4) = 0
x2 + y2 – 3x – 4y = 0

Answer. d

Right Marks: 3.00

Negative Marks: 1.00

40. Number of complex number(s) 'z' satisfying |z|n = |z|n–2 z2 + |z|n–2 z + 1 (n ∈ Q) and Re(z) ≠ is(are)

a. 0

b. 1

c. 2

d. 4

Solution. |z|n = |z|n–2(z2 + z) + 1 ...(1)


from (1) z2 + z is real


Since Re(z) ≠
⇒ ⇒ z is purely real
from (1) zn–1 + 1 = 0
⇒ z = –1 is only solution

Answer. b

Right Marks: 3.00


Negative Marks: 1.00

41. A polynomial function P(x) of degree 5 with leading coefficient one, increases in interval (–∞, 1) and (3, ∞) and decreases in interval (1, 3). Given
that P'(2) = 0 then value of P'(4) is

a. 240

b. 180

c. 120

d. 60

Solution. Degree of P(x) is 5 with leading coeff. one.


∴ degree of P'(x) is 4 with leading coeff. five.
Hence,
P'(x) = 5(x – 1)(x – 2)2(x – 3)
P'(4) = 5 × 3 × 4 × 1 = 60

Answer. d

Right Marks: 3.00

Negative Marks: 1.00

42. Let , then has the value equal to

a.

b.

c. π

d. –π

Solution. R3 → R3 – (cosx) R1

f(x) = (cos2x – secx – cot2x) (cosx – cos3x)

f(x) = cos3x(sin2x – 1) – sin2x


f(x) = –cos5x – sin2x

Answer. d

Right Marks: 3.00

Negative Marks: 1.00

Section: MATHEMATICS SEC-2 MATHEMATICS SEC-2 Instruction

43. If z is non-real complex number satisfying |z| = 2, then which of the following is(are) correct ?

a. (where z ≠ ±2)

b.
c. |z2 – 1| ≥ 3

d. |z2 – 1| ≤ 5

Solution. (A) For P(z) we get

For Q(z) we get

(B)

ΔAOB is equilateral Δ
∠AOB = and ∠AOB =

(C) |z2 – 1|2 = (z2 – 1)( – 1)


= z2 – z2 – + 1
|z|4 – 2(x2 – y2) + 1
= 17 – 2(x2 – y2) = 25 – 4x2
(∵ |z| = 2 ⇒ x2 + y2 = 4 where x ∈ [–2, 2])
9 ≤ |z2 – 1|2 ≤ 25
Hence 3 ≤ |z2 – 1| ≤ 5

Answer. a,c,d

Right Marks: 4.00

Negative Marks: 2.00

44. Let , where a, b, n are even natural numbers and Cr represent the coefficient of xr in the expansion of (1 +

x)100 then

a. a + b + n = 504

b. a + b = 402

c. a – n = 102

d. a + 2b + 3n = 900

Solution.

101 200C100 + 101 200C100 + 2100.2100


202 200C100 + 2200 = a 2nCn + 2b
a = 202; n = 100; b = 200

Answer. b,c

Right Marks: 4.00

Negative Marks: 2.00

45. Let ƒ(x) = 2tan–1x and g(x) be a differentiable function satisfying ∀ x, y ∈ R and g'(0) = 1 and g(0) = 2. The number of
integers 'x' satisfying ƒ2(g(x)) – 5ƒ(g(x)) + 4 > 0 where x ∈ (–10, 20) is greater than or equal to

a. 5

b. 6

c. 7

d. 8

Solution. g(x) = x + 2
f(g(x)) = 2tan–1(x+2) f2(g(x)) – 5f(g(x)) + 4 > 0
f(g(x)) < 1 or f(g(x)) > 4
tan–1(x + 2) < or tan–1(x + 2) > 2

as


Hence integers in range are {–9, –8, ..., –2}

Answer. a,b,c,d

Right Marks: 4.00

Negative Marks: 2.00

46. Let six children B1, B2, B3, B4, B5, B6 each have a fair 6 sided die which they roll simultaneously. Let E denotes the event that atleast three of
them roll a prime number and F denotes the event that B1, B2, B3 throw a prime number. If the conditional probability of F given E is equal to where
a and b are coprime then

a. 2a + b = 29

b. |a – b| = 10

c. ab ≤ 84

d. b – 4a = 0

Solution. P(Prime number) =

∴ ; ;n=6
P(E) = P(r ≥ 3)

∴ a = 4 & b = 21

Answer. a,c

Right Marks: 4.00

Negative Marks: 2.00

47. If pair of variable straight lines x2 + 4y2 + αxy = 0 (α is real parameter) cuts the ellipse x2 + 4y2 = 4 at two points A and B, then which of the
following cannot be the locus of point of intersection of tangents at A and B of ellipse

a. x2 + 4y2 = 0

b. 4x2 – y2 = 0

c. x2 – xy + 4y2

d. x2 – xy – 4y2 = 0
Solution. Let the point of intersection of tangents at A and B be P(h, k) then equation of AB is

....(1)
Homogenising ellipse with (1) we get

...(2)

Combined equation of OA and OB is


x2 + 4y2 + αxy = 0 ...(3)
Compare (2) and (3)

⇒ (h2 – 4) = 4(k2 – 1)
⇒ h2 – 4k2 = 0 ⇒ x2 – 4y2 = 0

Answer. a,b,c,d

Right Marks: 4.00

Negative Marks: 2.00

48. A non-zero vector is equally inclined to three vectors , , . Let be three vectors in plane of ; and
respectively then

a.

b.

c.

d.

Solution.
So are coplanar vectors
As is equally inclined to .
⇒ is perpendicular to .

Answer. b,c

Right Marks: 4.00

Negative Marks: 2.00

Section: MATHEMATICS SEC-3 MATHEMATICS SEC-3 Instruction

49. Let ƒ(x) = x3 – ax2 + bx + 6 where a, b ∈ {1, 2, 3, 4, 5, 6}. The probability that f(x) is strictly increasing is

Solution. f'(x) = 3x2 – 2ax + b ≥ 0 ∀ x ∈ R ⇒ (2a)2 – 4(3)(b) ≤ 0


a2 – 3b ≤ 0 This is possible for 16 ordered pairs (a, b)
(1,1), (1,2), (1,3), (1,4), (1,5), (1,6), (2,2), (2,3),
(2,4), (2,5), (2,6), (3,3), (3,4), (3,5), (3,6) & (4, 6) Hence probability

Answer. 0.44

Right Marks: 4.00

Negative Marks: 0.00


50. Let a1, a2, a3, ..., an be an increasing arithmetic progression of positive integers. If a3 = 13, then the maximum value of is

Solution.
a1 = 13 – 2d; a2 = 13 – d; a3 = 13; a4 = 13 + d; a5 = 13 + 2d
an = (13 – 2d) + (n – 1)d = (13 – 3d) + nd
= a13–2d + a13–d + a13 + a13+d + a13+2d

S = 5(13+10d)
Now d ≤ 6 as terms are positive
∴ Smax occurs at d = 6 and is equal to 365.

Answer. 365.00

Right Marks: 4.00

Negative Marks: 0.00

51. Let 'C' be the curve passing through the point (1, 1) has the property that the perpendicular distance of the normal from origin at any point P of
the curve is equal to distance of P from x-axis. If area bounded by curve 'C' and x-axis in first quadrant is kπ square units then k is equal to

Solution.

Equation of normal is
X + mY – (x + my) = 0 ...(1)
perpendicular distance from (0, 0) to equation (1) is

Put y2 = t ⇒

I.F. = e–ℓnx =

= –x + c ⇒ = –x + c
(1,1) lies on it so c = 2

C : x2 + y2 – 2x = 0
Area =
∴k= = 0.50

Answer. 0.50

Right Marks: 4.00

Negative Marks: 0.00

52. A circle is inscribed in a triangle with side lengths 3, 4, 5. A second circle interior to the triangle, is tangent to the first circle and to both sides of
the larger acute angle of triangle. The radius of the second circle is
Solution.

Radius of first circle

...(1)

Also sinC = ⇒

r = 0.38

Answer. 0.38

Right Marks: 4.00

Negative Marks: 0.00

53. Let f be a function defined by , where r = 3k, k ∈ I then value of is equal to

Solution. for r = 0

for r = 3

for r = 6

f(x) is periodic with period = 3

= 16.66 or 16.67

Answer. 16.66,16.67

Right Marks: 4.00

Negative Marks: 0.00

54. Let A = [aij] be a 3 × 3 matrix such that , then the value of det(adjA) is

Solution. aij = –aji ∀ i & j


So A is a skew-symmetric matrix of order-3
so det A = 0
∴ det(adjA) (det(A))2 = 0
Answer. 0.00

Right Marks: 4.00

Negative Marks: 0.00

Test Answer

1. (c) 2. (c) 3. (b) 4. (d) 5. (d) 6. (b) 7. (a,c,d) 8. (b,c,d) 9. (a,c) 10. (a,b,c)
13. (2.00) 14. (5.00) 15. (2.00) 16. (5.00) 17. (4.00) 18. (5.00)
11. (a,b,c) 12. (a,c,d) R_F:0.0000 R_F:0.0000 R_F:0.0000 R_F:0.0000 R_F:0.0000 R_F:0.0000 19. (b) 20. (a)
R_T:0.0000 R_T:0.0000 R_T:0.0000 R_T:0.0000 R_T:0.0000 R_T:0.0000
21. (b) 22. (b) 23. (a) 24. (a) 25. (b,c) 26. (a,b,c) 27. (a,b,c,d) 28. (a,b,d) 29. (a,b,c) 30. (a,b)
31. (10.00) 32. (3.00) 33. (2.50) 34. (19.00) 35. (2.00) 36. (2.00)
R_F:0.0000 R_F:0.0000 R_F:0.0000 R_F:0.0000 R_F:0.0000 R_F:0.0000 37. (d) 38. (c) 39. (d) 40. (b)
R_T:0.0000 R_T:0.0000 R_T:0.0000 R_T:0.0000 R_T:0.0000 R_T:0.0000
49. (0.44) 50. (365.00)
41. (d) 42. (d) 43. (a,c,d) 44. (b,c) 45. (a,b,c,d) 46. (a,c) 47. (a,b,c,d) 48. (b,c) R_F:0.0000 R_F:0.0000
R_T:0.0000 R_T:0.0000
53.
51. (0.50) 52. (0.38) 54. (0.00)
(16.66,16.67)
R_F:0.0000 R_F:0.0000 R_F:0.0000
R_F:16.6600
R_T:0.0000 R_T:0.0000 R_T:0.0000
R_T:16.6700

You might also like